Example of a linear subset of Hilbert space that is not closed

Click For Summary
A linear subset M of a Hilbert space is closed if and only if the set perpendicular to the perpendicular of M equals M. An example of a linear subset that is not closed is the set S defined by finite linear combinations of an orthonormal basis in an infinite-dimensional Hilbert space. Although sequences within S may converge, their limits can fall outside of S, demonstrating that S is not closed. The confusion arises from the assumption that convergence within S guarantees limits also reside in S, which is not the case in infinite dimensions. Understanding this distinction is crucial for grasping the properties of linear subsets in Hilbert spaces.
margaret37
Messages
12
Reaction score
0

Homework Statement



Prove that for a linear set M a subset of Hilbert space, that the set perpendicular to the set perpendicular to M is equal to M iff M is closed.

The Attempt at a Solution



I already have my proof -- but what is an example of a linear subset of H that is not closed?

I think I understand that we need an infinite dimensional Hilbert space -- that in a finite space M would have to be closed.

Thanks
 
Physics news on Phys.org
If H is a Hilbert space, and {e1,e2,...} is an orthonormal basis, then if you define S={a1*e1+a2*e2+...} where a finite number of the ai are nonzero, then S would be a subspace of H, right? But it wouldn't be closed.
 
Last edited:
Thank you for answering, unfortunately I'm still confused.

Why wouldn't it be closed? If I set up a sequence within S, wouldn't it necessarily converge to something also inside S.
 
margaret37 said:
Thank you for answering, unfortunately I'm still confused.

Why wouldn't it be closed? If I set up a sequence within S, wouldn't it necessarily converge to something also inside S.

p1=e1, p2=e1+e2/2, p3=e1+e2/2+e3/4, p4=e1+e2/2+e3/4+e4/8. Do you see where this is going? Aren't all of the pi in S? Isn't the limit in the Hilbert space, since H is a complete metric space?
 
Question: A clock's minute hand has length 4 and its hour hand has length 3. What is the distance between the tips at the moment when it is increasing most rapidly?(Putnam Exam Question) Answer: Making assumption that both the hands moves at constant angular velocities, the answer is ## \sqrt{7} .## But don't you think this assumption is somewhat doubtful and wrong?

Similar threads

  • · Replies 5 ·
Replies
5
Views
2K
Replies
2
Views
2K
  • · Replies 18 ·
Replies
18
Views
2K
  • · Replies 43 ·
2
Replies
43
Views
4K
  • · Replies 1 ·
Replies
1
Views
2K
Replies
2
Views
1K
  • · Replies 7 ·
Replies
7
Views
1K
Replies
34
Views
3K
  • · Replies 9 ·
Replies
9
Views
2K
Replies
1
Views
2K